A Strange Polynomial in 2015

Algebra Level 5

Let f ( x ) f(x) be a polynomial with integer coefficients such that

f ( 2 ) = f ( 0 ) = f ( 1 ) = f ( 5 ) = n f ( 2 ) = f ( 1 ) = f ( 5 ) = n f(2) = f(0) = f(1) = f(5) = n \\ f(-2) = f(-1) = f(-5) = -n

for some positive integer n n . Find the smallest possible value of n n .

Inspired from an AMC 10B problem.


The answer is 6300.

This section requires Javascript.
You are seeing this because something didn't load right. We suggest you, (a) try refreshing the page, (b) enabling javascript if it is disabled on your browser and, finally, (c) loading the non-javascript version of this page . We're sorry about the hassle.

1 solution

Steven Yuan
Feb 6, 2015

Let g ( x ) = f ( x ) n g(x) = f(x) - n . Some of the roots of g ( x ) g(x) occur when f ( x ) = n f(x) = n i.e. at x = 2 , 0 , 1 , 5. x = 2, 0, 1, 5. Thus, g ( x ) g(x) can be factored as x ( x 1 ) ( x 2 ) ( x 5 ) h ( x ) x(x-1)(x-2)(x-5)h(x) for some polynomial h ( x ) h(x) . If we plug in 2 , 1 , 5 -2, -1, -5 into f ( x ) n , f(x) - n, we get

f ( 2 ) n = g ( 2 ) = ( 2 ) ( 3 ) ( 4 ) ( 7 ) h ( 2 ) = ( 2 3 × 3 × 7 ) h ( 2 ) = 2 n f ( 1 ) n = g ( 1 ) = ( 1 ) ( 2 ) ( 3 ) ( 6 ) h ( 1 ) = ( 2 2 × 3 2 ) h ( 1 ) = 2 n f ( 5 ) n = g ( 5 ) = ( 5 ) ( 6 ) ( 7 ) ( 10 ) h ( 5 ) = ( 2 2 × 3 × 5 2 × 7 ) h ( 5 ) = 2 n \begin{aligned} f(-2) - n &= g(-2) = (-2)(-3)(-4)(-7)h(-2) \\ &= (2^3 \times 3 \times 7)h(-2) = -2n \\ \\ f(-1) - n &= g(-1) = (-1)(-2)(-3)(-6)h(-1) \\ &= (2^2 \times 3^2)h(-1) = -2n \\ \\ f(-5) - n &= g(-5) = (-5)(-6)(-7)(-10)h(-5) \\ &= (2^2 \times 3 \times 5^2 \times 7)h(-5) = -2n \\ \end{aligned}

Thus, n = ( 2 2 × 3 × 7 ) h ( 2 ) = ( 2 × 3 2 ) h ( 1 ) = ( 2 × 3 × 5 2 × 7 ) h ( 5 ) . -n = (2^2 \times 3 \times 7)h(-2) = (2 \times 3^2)h(-1) = (2 \times 3 \times 5^2 \times 7)h(-5). The smallest value of n n that satisfies this is the least common multiple of 2 2 × 3 × 7 2^2 \times 3 \times 7 , 2 × 3 2 2 \times 3^2 , and 2 × 3 × 5 2 × 7 2 \times 3 \times 5^2 \times 7 . This is 2 2 × 3 2 × 5 2 × 7 = 6300 . 2^2 \times 3^2 \times 5^2 \times 7 = \boxed{6300}.

For the sake of completeness, we show that a polynomial f ( x ) f(x) with our value of n n does exist. Plugging into our last equation, we get h ( 2 ) = 75 h(-2) = -75 , h ( 1 ) = 350 h(-1) = -350 , and h ( 5 ) = 6 h(-5) = -6 . Using Lagrange, we get h ( x ) = 63 x 2 464 x 751 h(x) = -63x^2 -464x - 751 . Thus,

f ( x ) 6300 = x ( x 1 ) ( x 2 ) ( x 5 ) h ( x ) = x ( x 1 ) ( x 2 ) ( x 5 ) ( 63 x 2 464 x 751 ) = 63 x 6 + 40 x 5 + 1890 x 4 1250 x 3 8127 x 2 + 7510 x , \begin{aligned} f(x) - 6300 &= x(x-1)(x-2)(x-5)h(x) \\ &= x(x-1)(x-2)(x-5)(-63x^2-464x-751) \\ &= -63x^6 + 40x^5 +1890x^4 -1250x^3 -8127x^2 +7510x, \end{aligned}

or f ( x ) = 63 x 6 + 40 x 5 + 1890 x 4 1250 x 3 8127 x 2 + 7510 x + 6300 f(x) = -63x^6 + 40x^5 +1890x^4 -1250x^3 -8127x^2 +7510x + 6300 .

(As pointed out by Kishore, "An easier way is to invoke a b f ( a ) f ( b ) a-b \mid f(a)-f(b) " is not immediately applicable.)

Note that for completeness, you have to show that such an integer polynomial exists. That is not immediately obvious.

Calvin Lin Staff - 6 years, 4 months ago

Log in to reply

Thanks. I'll do that.

Steven Yuan - 6 years, 4 months ago

Log in to reply

Hey Steven, did you just delete your question "beat your friend at subtraction" ?

Also Congrats on 150 Followers :)

A Former Brilliant Member - 6 years, 3 months ago

Log in to reply

@A Former Brilliant Member Thanks! :)

As for the problem, I have reposted it. I thought that the answer was incorrect at first, but then I realized it was ok. I have also edited the wording of the problem to make it clearer.

Steven Yuan - 6 years, 3 months ago

Log in to reply

@Steven Yuan Ok,I'll attempt the new question :)

A Former Brilliant Member - 6 years, 3 months ago

Then you get certain numbers, that divides 2 n 2n . But taking L . C . M L.C.M won't help. You should get 6300 6300 , but taking L . C . M L.C.M does not give that 9 n 9|n and 25 n 25|n . Please reply.

Kishore S. Shenoy - 5 years, 9 months ago

Log in to reply

Hm, that's right. We need the full force of Remainder Factor Theorem.

Calvin Lin Staff - 5 years, 9 months ago

Log in to reply

@Calvin Lin Remainder Factor Theorem? Can you explain... Instead if you use Lagrange Interpolation to get a 6th degree polynomial, you get the answer directly.

Kishore S. Shenoy - 5 years, 9 months ago

More or less the same method .Nice solution

A Former Brilliant Member - 6 years, 4 months ago

@Nikhil George , try this question! I challenge!

Kishore S. Shenoy - 5 years, 9 months ago

I think this is a partway application or proof of Lagrange Interpolation.

Kishore S. Shenoy - 5 years, 9 months ago

0 pending reports

×

Problem Loading...

Note Loading...

Set Loading...